1
$\begingroup$

Let $h:[0,1]\to[0,1]$ be a $\mathcal{C^1}$ function such that $h'(x)<0$ for all $x\in(0,1)$. I am trying to show that (not sure if it is true):

$$ \inf_{f\in \mathcal{H}}\left(\int_0^1\int_0^1 h(|x-y|)(f(x)-f(y))^2dxdy\right) < 2\int_0^1 h(x)dx $$ where $\mathcal{H} = \left\{f\in L^2([0,1]) : \int_0^1 f(t)dt = 0 \text{ and } \int_0^1 f^2(t)dt = 1\right\}$.

$\endgroup$

2 Answers 2

1
$\begingroup$

Note that \begin{equation} \int_0^1\int_0^1 h(|x-y|)(f(x)-f(y))^2\,dx\,dy-2\int_0^1 h(x)dx=2D_f(h), \end{equation} where \begin{equation} D_f(h):=\int_0^1 dx \int_x^1 dy\,h(y-x)(f(x)-f(y))^2-\int_0^1 h. \end{equation} So, it is enough to find, for each $h$ as in the OP, a function $f\in\mathcal H$ such that $D_f(h)<0$.

We shall do a bit more than this -- by letting $h$ be any continuous function on $[0,1]$ such that \begin{equation} h(0)>h(1). \end{equation}

Take any $t\in(0,1/2)$, and for all $x\in[0,1]$ let \begin{equation} f(x):=f_t(x):=\tfrac1{\sqrt{2t}}(I\{x<t\}-I\{x>1-t\}), \end{equation} where $I$ is the indicator. Then $f\in\mathcal H$. Also, \begin{equation} 2t(f(x)-f(y))^2=I\{x<t\}I\{y\ge t\}+I\{x\le1-t\}I\{y>1-t\}+2I\{x<t\}I\{y\ge1-t\} \end{equation} if $0<x<y<1$, whence, letting $t\downarrow0$, we have
\begin{equation} \int_0^1 dx \int_x^1 dy\, h(y-x)(f(x)-f(y))^2=\frac{J_1+J_2+2J_3}{2t}, \end{equation} where \begin{align} J_1&:=\int_0^t dx \int_t^1 dy\, h(y-x) \\ &= \int_0^t dx \int_{t-x}^{1-x} du\, h(u) \\ &= t\int_0^1 h- \int_0^t dx \Big(\int_0^{t-x}h+ \int_{1-x}^1 h\Big) \\ &= t\int_0^1 h- \int_0^t dx \Big(\int_0^{t-x}[h(0)+o(1)]+ \int_{1-x}^1 [h(1)+o(1)]\Big) \\ &= t\int_0^1 h- \frac{t^2}2\,[h(0)+h(1)+o(1)]; \end{align} similarly, \begin{align} J_2&:=\int_{1-t}^1 dy\,\int_0^{1-t} dx\, h(y-x) \\ &= t\int_0^1 h- \frac{t^2}2\,[h(0)+h(1)+o(1)]; \end{align} and \begin{align} J_3&:=\int_0^t dx \int_{1-t}^1 dy\, h(y-x) \\ &= t^2[h(1)+o(1)]. \end{align} Collecting all the pieces, we have \begin{equation} D_f(h)=\frac{J_1+J_2+2J_3}{2t}-\int_0^1 h=[h(1)-h(0)+o(1)]t/2<0 \end{equation} for small enough $t>0$ (depending on $h$). Thus indeed, for each continuous function $h$ on $[0,1]$ such that $h(0)>h(1)$, we have constructed a function $f\in\mathcal H$ such that $D_f(h)<0$.

$\endgroup$
2
  • $\begingroup$ Thanks @IosifPinelis for you answer. I am not sure if the function $g(x) = I_{\left\{x<1/2\right\}}- I_{\left\{x>1/2\right\}}$ is the solution for the problem. In fact if you take $h(x) = 1-x^2$, then $D_g(h) = 1/12>0$. I think the problem in your solution is in the calculation of $D_g(h_u)$, in fact for $u\in (0,1/2)$ your formula is correct. Nevertheless when $u\in (1/2,1)$ we have $D_g(h_u) = 2-4(1-u)^2-2u>0$! $\endgroup$
    – Samovem
    Mar 4, 2019 at 14:21
  • $\begingroup$ @Samovem : Thank you for your comment. I have fixed the error. $\endgroup$ Mar 5, 2019 at 6:48
0
$\begingroup$

Based on the previous comment, it is enough to find a function $f\in \mathcal{H}$ such that $$ \int\int_{|x-y| < u} (f(x)-f(y))^2 dxdy < 2u $$ for almost every $u\in (0,1)$.

$\endgroup$
1
  • $\begingroup$ I think this is conjecture is false. $\endgroup$ Mar 5, 2019 at 6:50

Your Answer

By clicking “Post Your Answer”, you agree to our terms of service and acknowledge you have read our privacy policy.

Not the answer you're looking for? Browse other questions tagged or ask your own question.